The program to control the entry of illegal drugs into the

This topic has expert replies
Moderator
Posts: 7187
Joined: Thu Sep 07, 2017 4:43 pm
Followed by:23 members

Timer

00:00

Your Answer

A

B

C

D

E

Global Stats

The program to control the entry of illegal drugs into the country was a failure in 1987. If the program had been successful, the wholesale price of most illegal drugs would not have dropped substantially in 1987.

The argument in the passage depends on which of the following assumptions?


(A) The supply of illegal drugs dropped substantially in 1987.

(B) The price paid for most illegal drugs by the average consumer did not drop substantially in 1987.

(C) Domestic production of illegal drugs increased at a higher rate than did the entry of such drugs into the country.

(D) The wholesale price of a few illegal drugs increased substantially in 1987.

(E) A drop in demand for most illegal drugs in 1987 was not the sole cause of the drop in their wholesale price.

OA E

Source: Official Guide

Legendary Member
Posts: 2214
Joined: Fri Mar 02, 2018 2:22 pm
Followed by:5 members

by deloitte247 » Sat Apr 20, 2019 10:51 pm

Timer

00:00

Your Answer

A

B

C

D

E

Global Stats

OPTION A:- INCORRECT
This isn't the assumption which the argument is operating on because the main concern of the argument is not the dropped prices of the illegal drugs but how a program made to control the entry of illegal drugs into the country failed.

OPTION B:- INCORRECT
The argument's main concern isn't the prices of the drugs, neither is there any certainty that the price, neither is there any certainty that the price paid by a average consumer of the illegal drugs didn't drop substantially in 1987 as opposed by the argument itself.

OPTION C:- INCORRECT
The argument didn't claim or give any indication or prove to show that the domestic production of illegal drugs increased. Although the argument's concern is about drugs being brought into the country, not the ones being produced in it. So, this cannot serve as an assumption to which the argument is based on.

OPTION D:- INCORRECT
This contradicts the argument which claims that the wholesale price of most illegal drugs dropped substantially in 1987. So, the argument isn't based on this assumption.

OPTION E:- CORRECT
The drop in demand for the most illegal drugs in wholesale price. This is true because another reason for the drop in price is the failed attempt of the program made to control the entry of illegal drugs as the argument claims. Therefore, this is the assumption that the argument runs on.